0 Daumen
547 Aufrufe

Hallo!

Ich soll hier überprüfen, ob das uneigentliche Integral konvergiert. Ich bin auf den Wert -2 gekommen, aber der Integralrechner zeigt eine andere Lösung und zwar: ln(-1)-2. Wieso?

Es gilt doch: ln |-1| = ln(1), da wir betragsstriche haben wir das ganze wieder positiv und ln(1) = 0. Dann kann als Lösung doch ja gar nicht ln(-1) rauskommen. Habe ich da einen Denkfehler? Bitte korrigiert mich, falls ich falsch liege.

Ich hab das ganze folgendermaßen gerechnet:

Aufgabe:

\( \begin{array}{l} \text { f) } \int \limits_{-1}^{1} \ln |x| d x=\lim \limits_{a \rightarrow-1^{+}} \int \limits_{a}^{1} \ln |x| d x= \\ \lim \limits_{a \rightarrow-1^{+}} \int \limits_{a}^{1} \frac{1}{u^{\top}} \frac{\ln |x|}{r} d x= \\ \mu=x \quad \mu^{\prime}=1 \\ v=\ln |x| \quad r^{\prime}=\frac{1}{x} \\ \lim \limits_{a \rightarrow-1^{+}}\left(x \cdot \ln |x|-\int x \cdot \frac{1}{x} d x=\right. \\ \left.\lim \limits_{a \rightarrow-1^{+}}(x \cdot \ln |x|-x)\right|_{a} ^{1}= \\ \lim \limits_{a \rightarrow-1^{+}}[(1 \cdot \ln |1|-1)-(a \cdot \ln |a|-a)]= \\ \lim \limits_{a \rightarrow-1^{+}}[-1+1 \cdot \ln |-1|-1]= \\ \lim \limits_{a \rightarrow-1^{+}}\left[-2+\frac{\ln (1)]=-2 \Rightarrow \text { konvergent }}{=0}\right. \end{array} \)




Lösung: \( \ln (-1)-2 \)

Avatar von

Der Integrand hat eine Singularität bei x=0. Daher muss man das Integral aufteilen in ein uneigentliches integral von-1 bis 0- und von 0+ bis 1.

Auch wenn ich es aufteile, wie dus erklärt hast, komme ich auf dasselbe Ergebnis.

Man kommt ja nie auf ln(-1), da ja die Zahl immer positiv ist, weil wir Betragsstriche haben.

Also so sieht das ganze dann aus, wenn ich die Aufteilung mache:

\( \begin{array}{l}\text { f) } \int \limits_{-1}^{1} \ln |x| d x=\int \limits_{-1}^{1} \underbrace{10}_{u^{\prime}} \underbrace{v}|x| d x \\ \lim \limits_{b \rightarrow 0^{-}} \int \limits_{-1}^{b} \ln |x| d x+\lim \limits_{a \rightarrow 0^{+}} \int \limits_{a} \ln |x| d x= \\ u=x \quad u^{\prime}=1 \\ v=\ln |x| \quad V^{\prime}=\frac{1}{x} \\ \lim \limits_{b \rightarrow 0^{-}}\left[x \cdot \ln |x|-\int x \cdot \frac{1}{x} d x\right]= \\ \lim \limits_{b \rightarrow 0^{-}}[x \cdot \ln |x|-x]_{-1}^{b}= \\ \lim \limits_{b \rightarrow 0^{-}}[(b \cdot \ln |b|-b)-(-1 \cdot \ln \mid-11+1) \\ =0 \ln |-1|-1=\ln (1)-1\end{array} \)

\( \begin{array}{l}\lim \limits_{a \rightarrow 0^{+}} \int \limits_{a}^{1} \ln |x| d x= \\ \lim \limits_{a \rightarrow 0^{+}}[x \cdot \ln |x|-x]_{a}^{1}= \\ {[1· \underbrace{\ln |1|-1-\underbrace{a \cdot \ln |a|}_{=0}+a]=-1}_{=0}} \\ \int \limits_{-1}^{1} \ln |x| d x=-1-1+\ln (1)=\ln (1)-2\end{array} \) 

= -2

Hallo

wie kommst du auf  \( \lim\limits_{a\to0} a*lna=0\) ? Das ist zwar richtig aber du solltest es zeigen!

wegen des |x| kannst du das Integral direkt als  2*\( \int\limits_{0}^1 ln(x)dx\) rechnen

lul

2 Antworten

+1 Daumen

Aloha :)

Wegen \(\ln|-x|=\ln|x|\) ist der Integrand symmetrisch zur \(y\)-Achse, daher können wir die Betragsstriche loswerden$$I=\int\limits_{-1}^1\ln|x|\,dx=2\int\limits_0^1\ln x\,dx=2\int\limits_0^1\underbrace{1}_{=u'}\cdot\underbrace{\ln x}_{=v}\,dx$$und danach partielle Integration anwenden:$$I=\underbrace{2\left[\underbrace{x}_{=u}\cdot\underbrace{\ln x}_{=v}\right]_0^1}_{=0}-2\int\limits_0^1\underbrace{x}_{=u}\cdot\underbrace{\frac1x}_{=v'}\,dx=-2\int\limits_0^1dx=-2\cdot[x]_0^1=-2$$

Avatar von 148 k 🚀

Diese Aufgabe nicht als uneigentliches Integral zu bearbeiten, halte ich für unprofessionell. Vgl. auch den Kommentar von lul.

Ich komme aber auch auf dasselbe Ergebnis wie Tschakabumba und lul.

Mathhilf, wie würdest dus genau rechnen? Könntest du deinen Rechenweg aufschreiben?

Ich habe Physik und Informatik studiert, bin also kein Mathe-Profi ;)

@ science:

Es sind ja alle Zutaten vorhanden: Du hast das Integral aufgeteilt, Du hast die kritische Stelle mit Grenzwerten angesetzt - was T ignoriert hat.

Bleibt nur noch luls Frage, ob man

\(a \ln(a) \to 0\)

Als allgemeines Wissen einstuft oder beweisen muss

Also stimmt dann meine Rechnung, die ich unter lul's Beitrag gepostet hab?

Ja, Dein Text ".... wenn ich die Aufteilung mache". Das ist richtig, allerdings:

Nochmal die Frage von lul: Ist Dir klar, dass 0* ln(0) nicht definiert ist?

Ich weiß nur, dass ln(0) undefiniert ist, also zu sagen, dass ln(0) = 0 wäre hier falsch. Da wir aber 0 x ln(0) haben, dachte ich, dass es 0 ist. Aber es ist ja 0, da wir ja mit 0 multiplizieren. Hätten wir hier keine 0, dann würde das gesamte Integral divergieren

Wenn \(f(x)=xg(x),x>0\), dann heißt das nicht automatisch \(f(x) \to 0\). Gegenbeispiel: \(g(x)=1/x^2\).

Wir haben es hier mit einem Fall vom Typ \(0 \cdot \infty\) zu tun

Ich hab`s jetzt erneut ausgerechnet, hab die Fallunterscheidung auch gemacht und so sieht‘s nun aus:

\( \begin{array}{l}\text { f) } \int \limits_{-1}^{1} \ln |x| d x=\int \limits_{-1}^{0} \ln |x| d x+\int \limits_{0}^{1} \ln |x| d x {\infty}  \\ \text { 1.Fall: } x<0 \Longleftrightarrow \ln (-x) \\ \text { 2. Fall: } x \geqslant 0 \Leftrightarrow \ln (x) \\ \lim \limits_{b \rightarrow 0^{-}} \int \limits_{-1}^{b} 1 \ln (-x) d x \\ \mu=-x \\ \frac{d u}{d x}=-1 \\ d u=-d x \\ \lim \limits_{b \rightarrow 0^{-}} \int \limits_{-1}^{b} 1 \cdot \ln (u)(-d u)=\lim \limits_{b \rightarrow 0^{-}}-\int \limits_{-1}^{b} 1 \cdot \ln (u) d u \\ d x=-d u \\\end{array} \)

\( \begin{array}{l}f=-u \quad f^{\prime}=-1 \\ g=\ln (u) \quad g „=\frac{1}{u} \\ \left(-u \cdot \ln (u)-\int-u \cdot \frac{1}{u} d u\right. \\ -u \cdot \ln (u)+u=[x \cdot \ln (-x)-x]_{-1}^{b} \\ \lim \limits_{b \rightarrow 0^{-}}[b \cdot \ln (-b)-b+1 \cdot \ln (1)-1]= \\ =0-0+0-1=-1 \\ \lim \limits_{a \rightarrow 0^{+}} \int \limits_{a}^{1} \ln (x) d x=\lim \limits_{a \rightarrow 0^{+}}\left[x \cdot \ln (x)-\int x \cdot \frac{1}{x} d x\right. \\ u=x \quad u^{\prime}=1 \\ v=\ln (x) \quad \gamma^{\prime}=\frac{1}{x} \\ \lim \limits_{a \rightarrow 0^{+}}[x \cdot \ln (x)-x]_{a}^{1}= \\ \lim \limits_{a \rightarrow 0^{+}}[1 \cdot \ln (1)-1-a \cdot \ln (a)+a]= \\ 0-1-0 \cdot(-\infty)+0=-1 \\ \int \limits_{-1}^{1} \ln |x| d x=-1-1=-2 \\\end{array} \)

Könnte wer einen Blick werfen? Kann ich das so stehen lassen? Könnte mir bitte jemand eine Rückmeldung geben?

Das sieht schon sehr gut aus, aber du schreibst quasi:$$\lim\limits_{x\to0}\left(x\cdot\ln x\right)=0\cdot(-\infty)=0$$Dann wäre z.B. auch:$$\lim\limits_{x\to0}\left(1\right)=\lim\limits_{x\to0}\left(x\cdot\frac1x\right)=0\cdot\infty=0$$

Erkennst du das Problem? Du kannst nicht sagen, dass \(0\cdot\infty\) gleich \(0\) ist.

Streng genommen kannst du den Grenzwert z.B. mit L'Hospital bestimmen:$$\lim\limits_{x\to0}\left(x\cdot\ln x\right)=\lim\limits_{x\to0}\left(\frac{\ln x}{\frac1x}\right)\stackrel{(\text{L'Hospital)}}{=}\lim\limits_{x\to0}\left(\frac{\frac1x}{-\frac{1}{x^2}}\right)=\lim\limits_{x\to0}(-x)=0$$

Danke für deine Rückmeldung :)

Aber muss ich da unbedingt L'Hospital anwenden? Gibt's da eine andere Variante auch ohne mit L'Hospital den Grenzwert zu berechnen? Und leider erkenne ich das Problem hierbei noch nicht ganz. Warum kann ich nicht sagen, dass \(0\cdot\infty\) gleich 0 ist?

Das mit L'Hospital habe ich nun vestanden, man darf die Regel nur dann anwenden, wenn 0 * unendlich oder \( \frac{unendlich}{unendlich} \) oder \( \frac{0}{0} \) haben.

Was noch unklar ist: soll ich überall lim (a--> 0+) und lim (b-->0+) hinschreiben, da wir uns ja immer von rechts annähern? Dann darf ich ja den linksseitigen Grenzwert, also lim (b → 0-) nicht hinschreiben

0 Daumen

Weg ohne partielle Integration:

\( \int\limits_{-1}^{1}ln|x|dx=2*\int\limits_{0}^{1}ln(x)dx \)

\(e^x\) ist die Umkehrfunktion von  \(ln(x)\)

\(A= \int\limits_{-∞}^{0}e^xdx=1 \)

Avatar von 36 k

Was ist A? Was ist A?

\(A \)    soll die Fläche sein.

Aber molietis ich habe unter Tschakabumbas Beitrag die Aufgabe erneut gerechnet und bin auf ein anderes Ergebnis gekommen. Passt mein Ergebnis nicht?

Ein anderes Problem?

Stell deine Frage

Willkommen bei der Mathelounge! Stell deine Frage einfach und kostenlos

x
Made by a lovely community